Docsity
Docsity

Prepare for your exams
Prepare for your exams

Study with the several resources on Docsity


Earn points to download
Earn points to download

Earn points by helping other students or get them with a premium plan


Guidelines and tips
Guidelines and tips

Summary of Lecture Notes in Health Assessment, Study notes of Health sciences

This is a complete and detailed summary lecture note in health assessment

Typology: Study notes

2017/2018

Available from 02/22/2022

van-keen
van-keen 🇵🇭

5 documents

1 / 44

Toggle sidebar

Often downloaded together


Related documents


Partial preview of the text

Download Summary of Lecture Notes in Health Assessment and more Study notes Health sciences in PDF only on Docsity! Now is the best time to be the person you dream of becoming P a g e | 1 HEALTH ASSESSMENT E-HANDOUTS (CBRC) 1 CBRC E-Hand Out: HEALTH ASSESSMENT Nurse Licensure Examination Subject Fundamentals of Nursing Topic Health Assessment Subtopic Initial Data Gathering Objective After reading this, you will be able to: a. Determine the steps in in initial data assessment of clients seeking care b. Develop the standard skills in communicating and establishing the nurse-patient relationship Discussion What is Health Assessment? -Health Assessment is the data gathering of an individual in all aspects including the spiritual and emotional state. What is the nurse’s role in health assessment? -Advocate A nurse advocate is a nurse who works on behalf of patients to maintain quality of care and protect patient’s rights. They intervene when there is a care concern, and following the proper channels, work to resolve any patient care issues. Steps in Data Assessment 1. Establish Rapport 2. Obtain Health History 3. Assess level of consciousness 4. Gather equipment needed 5. Perform Hand washing Chief Complaint In your patient’s own words, document the chief complaint. The chief complaint may be elicited by asking one of the following questions: • So, tell me why you have come here today? • Tell me what your biggest complaint right now? • What is bothering you the most right now? • If we could fix any of your health problems right now, what would it be? • What is giving you the most problems right now? If your patient has more than one complaint, discuss which one is the most troublesome for them and document the complaints in order of importance as determined by the patient (Jarvis,2012) Present Health Status Obtaining information about the present health status allows the nurse to investigate current complaints. The mnemonic PQRST, utilizes a structured format for information gathering, including evaluation of pain, and provides an efficient methodology to communicate with other healthcare providers. Use PQRST to assess each symptom and after any intervention to evaluate any changes or response to treatment (Jarvis,2012). PQRST P= Provocative or Palliative-What makes the symptom(s) better or worse? Q= Quality-Describe the symptom(s). R= Region or Radiation -Where in the body does the symptom occur? Is there radiation or extension of the symptom(s) to another area of the body? S= Severity -On a scale of 1-10, (10 being worst) how bad is the symptom(s)? Another visual scale may be appropriate for patients that are unable to identify with this scale. T= Timing -Does it occur in association with something else (i.e. eating, exertion, movement)? Past Health History It is important to ask questions about your patient’s past health history. The PHH should elicit information about the patient’s childhood illnesses and immunizations, accidents or traumatic injuries, hospitalizations, surgeries, psychiatric or mental illnesses, allergies, and chronic illnesses. For women, include history of menstrual cycle, how many pregnancies and how many births (Jarvis,2012). Family History Now is the best time to be the person you dream of becoming P a g e | 2 Current Health Status: Information collected should also include details about your patient’s personal habits such as smoking or drinking, nutrition, cholesterol, and if there is a history of heart disease or hypertension. Medications: Obtain a list of current medications, including dose and frequency, as well as reason for taking them. Remember to ask the patient about over the counter medications, vitamins, and herbal supplements (Jarvis, 2012). HEALTH ASSESSMENT DREX Notes (CBRC) Nurse-Patient Relationship = PROFESSIONAL Characteristics: “HHH” = Head (Cognition), Hands (Skills), Heart (Attitude) Goal: INDEPENDENCE OF THE CLIENT FOCUS (ROLE): “ROLE mo then HANDWASH VS” ➢ Rapport, Obtaining Health History, Level of Consciousness, Equipment Hand washing & Vital Signs (VS) HEALTH HISTORY ASSESSMENT (SAMPLE): ➢ Symptoms (FELT by the patient) / Signs (OBSERVED by the nurse/doctor), Allergies, Medications, Past Medical History, Last Oral Intake, Events leading to the illness or injury I. RAPPORT ➢ Reassurance, Active Listening, Posture: Gesture – affect, Privacy & Empathy , Observe Layman’s term, Respect Culture, Touch II. Types of Touches: a. Functional (BEST for patients) = PROFESSIONAL b. Social = POLITE c. Friendship = WARMTH d. Love = INTIMACY e. Sexual = AROUSAL III. HEALTH HISTORY ➢ PRIMARY DATA: PATIENT ➢ SECONDARY DATA: FAMILY or OTHER SUPPORT PERSONS, HEALTHCARE PROFESSIONALS, RECORDS AND REPORTS, LABORATORY AND DIAGNOSTIC ANALYSES AND RELEVANT LITERATURES. PAST, PRESENT, FAMILY, FUTURE IV. LEVEL OF CONSCIOUSNESS (LOC) 1st Level: ALERT (Awake and Conversant without tangentiality, circumstantiality, and looseness of associations) 2nd Level: CONFUSION (Unclear thoughts) 3rd Level: DISORIENTATION (Cannot recall the place, date and time) 4th Level: LETHARGIC (Responsive upon VERBAL approach) 5th level: OBTUNDED (Responsive upon TACTILE approach) 6th Level: STUPOR (Responsive upon PAIN is inflicted) 7th Level: COMA (Not responsive) 1. CONSTRICTED EYES: Drugs taken: ➢ Heroin, Morphine, Oxycodone, Fentanyl, Methadone, Codeine, Hydrocodone 2. DILATED EYES: Drugs taken: ➢ Amphetamines, Methamphetamines, Cocaine or Crack, Hallucinogens (i.e. LSD or mushrooms), Opiates (prescription painkillers), Heroin, Marijuana, Speed 3. RED EYES: Drugs taken: ➢ Marijuana, Cocaine or Crack, Benzodiazepines (i.e. Xanax), Depressants (i.e. Alcohol or Sedatives) SIGN OF IMPENDING DEATH: Constant dilation of pupils V. HAND WASHING 1. MEDICAL: ➢ Uses SOAP, Not less than 10 seconds ➢ Recommendation: 20 seconds or more 2. SURGICAL ➢ Uses STERILIUM and BETADINE, Not less than 5 minutes Now is the best time to be the person you dream of becoming P a g e | 5 ➢ CHRONEMICS = TIME ➢ PARALANGUAGE = VOICE ➢ KINESICS = BODY LANGUAGE BODY TYPES ➢ ECTOMORPH = Skinny; Difficulty in gaining weight ➢ MESOMORPH = Naturally muscular; easy gain and loses weight ➢ ENDOMORPH = Round; Difficulty in losing weight; slower metabolism BODY SHAPES ➢ RECTANGLE, TRIANGLE, HOURGLASS, INVERTED TRIANGLE, ROUND PHYSICAL EXAMINATION A. INSPECTION ➢ Sight & Smell ➢ FACTORS: a) Positioning b) Lighting (Visualization) − for EYE Assessment (PERRLA and VISION):  SIZE of NORMAL PUPIL = 3-7 mm ✓ MYDRIASIS - >7 mm; Dilated ❖ give MYOPIC DRUGS! ✓ MIOSIS ❖ constricted ✓ ANISOCORIA - unequal pupils  SHAPE = EQUALLY round  NORMAL REACTION TO LIGHT − Constriction = well lit area − Dilation = dim or dark area  NORMAL REACTION TO ACCOMODATION (NEAR OBJECT) - Constriction  DIRECT RESPONSE − If the light or object is shone in the right eye, the right pupil constricts.  CONSENSUAL RESPONSE − If the light or object is shone produces constriction on the right pupil, the left pupil also constricts.  LEGAL BLINDNESS = 20/200  MYOPIA = Nearsightedness (CONCAVE)  HYPERROPIA = Farsightedness (CONVEX)  DIM LIT ROOM IS USED  SNELLEN CHART (Standard)  RosenBaum CHART (14 inches away from the eyes). c) Exposure − Expose ONLY the body part that is to be currently examined. d) Comparison ➢ Before INSPECTION: a. Establish Rapport b. Good Lighting c. Consent ➢ IAPePa (“I Am Peter Parker”) = ABDOMEN Reasons: ✓ To avoid alteration of assessment bowel sounds ✓ To detect presence for BRUITS (signifies AAA)  NRSNG MGT: Put sign “DO NOT PALPATE!” ➢ IPaPEA = GENERAL B. PALPATION ✓ TYPES: ❖ LIGHT PALPATION − Surface, Parallel alignment of hand to body surface. Now is the best time to be the person you dream of becoming P a g e | 6 ❖ DEEP PALPATION − 1-3 inches (2.5-7.5 cm) deep, Delicate procedure, Requires supervision ➢ FINGER PADS = Fine Discrimination ➢ PALMAR/ULNAR = Thrills, Vibrations, Fremitus − To know what size, shape or texture of a body area or organ. ➢ DORSAL = Temperature • Other Purposes for PALPATION: Temperature, Texture (Smoothness and roughness of the skin), Moisture, Organ size and location, Rigidity or Spasticity, Vibration, Position, Size, Presence of lumps or masses C. PERCUSSION − Sense of hearing is used − Striking approach of a surface ➢ TYPES: a. Direct (uses Plexor Dominant/Dominant hand) -striker b. Indirect (uses Pleximeter/Non-dominant Hand) -Bimanual − Being strike on the interphalangal joints. ➢ SOUNDS: a. Resonant (air filled lungs) - Hollow, low-pitched sounds b. Hyperresonance (colds, emphysematous lung – hyperinflated – too much air inside alveolar sacs of the lungs) - Booming, louder-pitched sounds - NORMAL for PEDIATRIC LUNG. c. Tympanic (air filled Stomach) - Drum-like d. Dull (Diaphragm, heart, liver, spleen) - “thud-like” , Dense tissue e. Flat (Bones, muscles, tumors) - Extremely dull (due to highly dense tissue) D. AUSCULTATION TYPES: ➢ DIRECT AUSCULTATION - uses unaided ear (not using an instrument) ➢ INDIRECT AUSCULTATION - uses instrument (stethoscope) For STETHOSCOPE: ➢ belL = LOW PITCH sounds - For BRUITS, ABNORMAL HEART SOUNDS (S3 and S4), BLOOD PRESSURE ➢ diapHragm = HIGH PITCH sounds - For NORMAL and ABNORMAL BOWEL SOUNDS LISTENING to sounds produced by the body: ✓ Heart, Blood Vessels, Lungs, Abdomen BODY MASS INDEX ➢ Measurement of body fat based on weight and height ➢ BMI = WEIGHT (kgs)/ HEIGHT (m2) ➢ Lbs/2.2 = ____kg BMI CATEGORIES SEVERELY UNDERWEIGHT <16 UNDERWEIGHT 16-18.4 NORMAL 18.5-24.9 OVERWEIGHT 25-29.9 OBESE >30 VITAL SIGNS A. TEMPERATURE ➢ Temperature center= HYPOTHALAMUS ➢ ROUTES: a) ORAL = MOST ACCESSIBLE and MOST CONVENIENT Contraindications: Vomiting & Seizure b) RECTAL = MOST ACCURATE (core body) Contraindications: Diarrhea, Hemorrhoids, Cranial Nerve 10 c) TYMPANIC = FASTEST (2-3 seconds) Contraindications: Otitis Media d) AXILLA = LEAST ACCURATE; SAFE Bath for 30 minutes ➢ CLEANING THERMOMETERS Now is the best time to be the person you dream of becoming P a g e | 7 ❖ BEFORE USING: BULB to STEM ❖ AFTER USING: STEM to BULB ➢ TYPES OF FEVER a) Remittent = Wide Fluctations (above normal temp.) b) Constant = Minimal Fluctuations (above normal temp.) Ex. 39.1 – 39.4 – 39.7 c) Intermittent = On and Off Fever Ex. 39.1 – 39.4 – 38.5 d) Relapsing = 1-2 days of short febrile episode B. PULSE ➢ NORMAL: 60-100 bpm ➢ BRADYCARDIA = <60 bpm ➢ TACHYCARDIA = >100 bpm ➢ RADIAL PULSE is felt on the wrist, just under the thumb. ➢ APICAL PULSE (Point of Maximal Impulse – PMI) LOCATION: 5th Intercostal space at the left midclavicular line. ➢ SITES (6 PULSE POINTS/SITES (CBARFP) – font color RED): a) Temporal, Carotid, Brachial, Apical, Radial, Femoral, Posterior Tibial, Popliteal, Pedal ➢ PULSE DEFICIT ✓ Apical – Peripheral ✓ NORMAL = 0 ➢ PULSE VOLUME a) +0 = ABSENT b) +1 = THREADY/WEAK c) +2 = NORMAL d) +3 = ABOVE NORMAL e) +4 = BOUNDING ➢ PULSE OXIMETER ✓ NORMAL = 95-100% ✓ THREATENING = <70% ✓ SITES: FINGERS, TOES, EARLOBES, NOSETIPS, FOREHEAD – Use of PATCHES C. RESPIRATION ➢ NORMAL: 12-20 bpm ➢ BRADYPNEA = <12 bpm ➢ TACHYPNEA = >20 bpm ➢ (-) APNEA = Cessation of Breathing ➢ Difficulty of Breathing (DOB) = Dyspnea ➢ ONE CYCLE (1 RESPIRATION) = 1 INSPIRATION THEN 1 EXPIRATION Muscles of Inspiration Muscles of Expiration A. Accessory A. Quiet Breathing Sternocleidomastoid (elevates sternum) Expiration results from passive, elastic recoil of the lungs, rib cage and diaphragm. Scalenes Group (elevates upper ribs) B. Active Breathing C. Principal Internal intercostals EXCEPT interchondral part (pull ribs down) External intercostals – interchondral part of internal intercostals (elevates ribs) Abdominals (pull ribs down, compress abdominal contents, thus, pushes diaphragm up) Diaphragm (dome descends, thus increasing vertical dimension of thoracic cavity; elevates lower ribs) Quadratus Lumborum (pull ribs down) ➢ NORMAL BREATH SOUNDS: ❖ BRONCHIAL - High-pitched sounds that is heard over the tracheobronchial tree. ❖ BRONCHOVESICULAR - Moderate-pitched sounds that are heard over the bronchioles. Now is the best time to be the person you dream of becoming P a g e | 10 G. MUSCULOSKELETAL ✓ ROM TEST ❖ 5 = Full ROM with against gravity, full resistance ❖ 4 = Full ROM with against gravity, some resistance ❖ 3 = Full ROM with against gravity, some resistance ❖ 2 = Full ROM with against gravity, Gravity is eliminated ❖ 1 = Small-flickering contraction ❖ 0 = No contraction ✓ Inflammation is characterized by 5 cardinal signs: 1) RUBOR (REDNESS) 2) CALOR (INCREASED HEAT) 3) TUMOR (SWELLING) 4) DOLOR (PAIN) 5) FUNCTIO LAESA (LOSS OF FUNCTION) REFLEXES 0 NO RESPONSE 1 DIMINISHED 2 NORMAL 3 ABOVE NORMAL; BRISK 4 HYPERACTIVE ✓ REFLEXES: ❖ DEEP TENDON REFLEXES: -Biceps Reflex (C5/C6), Brachioradialis Reflex (C6), Triceps Reflex (C7), Patellar Reflex (L4), Achilles Tendon (S1) ❖ PLANTAR RESPONSE ❖ REFLEX TESTED IN SPECIAL SITUATIONS -Spinal cord injury, Frontal release signs, Posturing ❖ SCALE: 0 ABSENT 1+ HYPOACTIVE 2+ NORMAL 3+ HYPERACTIVE 4+ HYPERACTIVE WITH CLONUS 5+ SUSTAINED CLONUS ✓ GALLEAZZI TEST (ALLIS SIGN) - A test to assess hip displacement (Congenital or injury) - Unequal knee length ✓ TYPES OF CLUBFOOT:  Talipes Equinovarus (CLUB FOOT) - Internal rotation of foot and contracted Achilles tendon (Plantar flexion). -MOST COMMON: Talipes Varus, Talipes Valgus, Talipes Equinus, Talipes Calcaneus H. INTEGUMENTARY ✓ PARTS: Hair, Nails , Skin ✓ LAYERS OF SKIN: ❖ Epidermis - contains cells (melanin) that produce pigment and protect immune system. - Avascular (absence of blood vessels) - PARTS: COLUGRASPIBA ❖ Stratum corneum, Stratum lucidum, Stratum granulosum, Stratum spinosum, Stratum basale Now is the best time to be the person you dream of becoming P a g e | 11 ❖ Dermis - Contains nerve endings, oil and sweat glands, and hair follicles. -Vascular - PARTS: Papillary & Reticular ❖ Subcutaneous Tissue - Made up of fat, connective tissue, and larger blood vessels. ✓ VITILIGO - Long-term condition where pale white patches develop on the skin. ✓ ALBINISM- Group of inherited disorders that result in little or no production of the pigment melanin, which determines the color of the skin, hair and eyes. ✓ BIRTHMARKS- are marks that result of excessive accumulations of melanin which is present at birth or appears shortly after birth. ✓ SKIN CHANGES ❖ Pallor- Unusual lightness of skin color - may be caused by reduced blood flow and oxygen or by a decreased number of red blood cells. ❖ Cyanosis- Bluish color to the skin or mucous membrane is usually due to a lack of oxygen in the blood. ❖ Jaundice- Yellow staining of the skin and sclerae (the whites of the eyes) by abnormally high blood levels of the bile pigment bilirubin. ❖ Erythema- Redness of the skin that results from capillary congestion. - can occur with inflammation, as in sunburn and allergic reactions to drugs. ✓ SKIN LESIONS ❖ PRIMARY a) Macule- Flat, distinct, discolored area of skin less than 1 centimeter (cm) wide. b) Papule- Solid, elevated lesion with no visible fluid which may be up to ½ cm. in diameter. c) Vesicle- A small fluid-filled blister on the skin. d) Pustule- a small collection of pus in the top layer of skin (epidermis) or beneath it in the dermis. e) Urticaria- Raised, itchy areas of skin that are usually a sign of an allergic reaction. f) Bullae- A fluid-filled sac or lesion that appears when fluid is trapped under a thin layer of your skin. ❖ SECONDARY a) Crust- are dried sebum, pus, or blood usually mixed with epithelial and sometimes bacterial debris. b) Ulcer- A break in skin or mucous membrane with loss of surface tissue, disintegration and necrosis of epithelial tissue, and often pus. c) Necrosis- The death of body tissue. d) Erosion- Loss of some or all of the epidermis (the outer layer) leaving a denuded surface. e) Skin atrophy- The degeneration and thinning of the epidermis and dermis. f) Fissure- is a crack or tear in the skin. g) Lichenification- Thick, leathery skin, usually the result of constant scratching and rubbing. ❖ SPECIAL a) Purpura- Purple-colored spots and patches that occur on the skin, and in mucus membranes, including the lining of the mouth. b) Telangiectasia- Dilatation of small blood vessels (arterioles, capillaries, venules), often multiple in character. c) Comedone- The primary sign of acne, consisting of a widened hair follicle filled with keratin skin debris, bacteria, and sebum (oil). ✓ NAILS NORMAL ABNORMAL CONVEX SHAPE CLUBBING OF FINGERNAILS − Caused by lack of oxygenation and heart problems. DEGREE OF ANGLE KOILONYCHIA (spoon nails) - refers to abnormally thin nails (usually of the hand) which have lost their convexity, becoming flat or even concave in shape. - a sign of hypochromic anemia, especially iron-deficiency anemia. ✓ MOUTH ❖ TEETH: ▪ NORMAL NO. OF ADULT TEETH = 32 ▪ DECIDUOUS = 20 ❖ ABNORMALITIES ▪ GINGIVITIS - Inflammation of the gums. Now is the best time to be the person you dream of becoming P a g e | 12 ▪ GLOSSITIS - Inflammation of the tongue. ▪ STOMATITIS - Mouth sores ▪ MACROGLOSSIA - Abnormal enlargement of the tongue ✓ THORAX  NORMAL AP Diameter = 2:1 ABNORMALITIES ANTERIOR POSTERIOR BARREL CHEST - air retention/ air trapping - Emphysema (PINK PUFFERS) patients KYPHOSIS - Excessive outward curvature of the spine - In CPR, kyphosis patients use donut pillows. PIGEON CHEST - Pectus carinatum - caused by Marfan Syndrome or rickets. LORDOSIS - Forward curvature of the spine. - Commonly seen in pregnant women. FUNNEL CHEST - Pectus excavatum - caused by Marfan Syndrome or rickets. SCOLIOSIS - Lateral curvature of the spine. HEALTH ASSESSMENT: DIAGNOSTIC TEST NOTES 1. Based on the paradigm of nursing, health refers to the holistic level of wellness that the person experiences. Sniper identifies the nursing domain in a paradigm which includes: ANSWER: Person, situation, environment, nursing 2. Primordial prevention focuses on preventing the emergence of risk factors. Primary prevention aimed at health promotion and includes: ANSWER: Immunization is PRIMARY prevention. Cancer screening is SECONDARY prevention. Self-administration of steroid is an example of TERTIARY prevention. 3. To check if a client has a possible Vitamin C deficiency, or scurvy, the nurse must make sure to examine the client’s: ANSWER: Gingiva. Scurvy is a disease characterized by soft, bleeding gums (gingivitis); along with loose teeth, pinpoint hemorrhages, muscle and joint pain and poor wound healing. 4. Specific prevention focuses on removing or reducing the levels of the risk factors. Mrs. Assassin was scheduled for Cervista Test by Nurse Sage. This action is an example of: ANSWER: Cervista Test is an example of SECONDARY prevention. It focuses on early identification of health problems and prompts intervention to alleviate health problems. Its goal is to identify individuals in an early stage of disease process and to limit future disability. 5. According to Florence Nightingale, health is a state of being well and using every power the individual possesses to the fullest extent. Which of the following individuals appear to have taken on the sick role? ANSWER: An employer who is ill and says “I won’t be able go to the office today.” 6. The World Health Organization defines health as the state of complete physical, mental, and social well-being, and not merely the absence of disease. During which stage of illness will we expect Rogue to relinquish the dependence role? ANSWER: Recovery or Rehabilitation During the Recovery and Rehabilitation stage, the client is expected to relinquish the dependent role and resume former roles and responsibilities. 7. Personal responsibility and sense of control are the key concepts for promotion of health. As the nurse reviews the client’s level of knowledge after several health-teaching sessions, she determined that the client still fails to follow the information provided. The nurse must respond by: Now is the best time to be the person you dream of becoming P a g e | 15 31. An oxygen delivery system is prescribed for a client with Chronic Obstructive Pulmonary Disease (COPD) to deliver a precise oxygen concentration. Which of the following types of oxygen delivery systems would Nurse Sarah anticipates to be prescribed? ANSWER: Venturi mask This device uses different size adaptors to deliver a fixed or predicted FiO2. The FiO2 delivered depends on the flow rate and/or entrainment port size. It is used for patients who have COPD when an accurate FiO2 is essential and carbon dioxide buildup must be kept to a minimum. Humidifiers usually are not used with this device. ( 60% to 100%) 32. Nurse Sarah is caring for a client with emphysema. The client is receiving oxygen. Nurse Sarah assesses the oxygen flow rate to ensure that it does not exceed: ANSWER: 2L/min O2 therapy may be prescribed but must be used cautiously. The goal of O2 therapy is to maintain the PaO2 between 50 and 60 mmhg. The initial liter flow is usually 1-3L/min. 33. A group of nursing students is discussing about the descriptions related to non-rebreather mask. The student incorrectly states the proper description of non-rebreather mask to nurse Sarah if she identified which of the following? ANSWER: “The valves should open during inhalation and close during exhalation”. With Non-rebreather masks, make sure valves are open during expiration and closed during inhalation to prevent drastic decrease in FIO2 (fraction of inspired oxygen) 34. Madeleine Leininger proposed that nursing is learned humanistic and scientific profession and discipline which is focused on the human care phenomena and activities in order to support, facilitate, or enable individuals or groups to maintain or regain their well-being (Transcultural Nursing). Who among the following theorists conceptualized the framework for psychiatric nursing, wherein a nurse must established a therapeutic relationship with the client? ANSWER: Hildegard Peplau Hildegard Peplau is known for her theory on Interpersonal Relations in Nursing. Betty Neuman is known for the Health Care Systems Model. Imogene King is known for the Goal Attainment Theory Lydia Hall is known for the Care-Core-Cure Model 35. Which nursing action by Nurse Sarah is essential to prevent hypoxemia during tracheal suctioning on her patient? ANSWER: Administering 100% oxygen to reduce the effects of airway obstruction during suctioning. Hyperoxygenation is performed bt increasing the intake of oxygen immediately prior to suctioning and when appropriate after suctioning (Pedersen et al., 2008) and helps reduce the occurrence of hypoxemia. Hyper oxygenation (pre-oxygenation) before suctioning offers some protection from a drop in arterial blood oxygen. 36. A client requires tracheal suctioning through the nose. Which nursing action by Nurse Sarah would be incorrect? ANSWER: Suctioning for 20 seconds 37. In verifying and ensuring the placement of an endotracheal tube, Nurse Sarah must know that the following are necessary EXCEPT: ANSWER: Use maximal occlusive pressure when inflating the cuff in order to create a seal Verifying Tube Placement. Verify the distal tip marking on endotracheal tube and immediately after ET tube is inserted, placement should be verified. The most accurate ways to verify placement are by checking end-tidal carbon dioxide levels and by chest x-ray. Assess for breath sounds bilaterally, sound over the gastric area, symmetric chest movement, and air emerging from ET tube. Auscultate over the trachea for presence of air leak. 38. Nurse Sarah is caring for a client immediately after removal of the endotracheal tube following radical neck dissection. Nurse Sarah reports which of the following signs immediately if experienced by the client? ANSWER: Stridor Auscultate breath sounds as needed. In the immediate postoperative period, place stethoscope over the trachea to assess for Stridor. Abnormal breath sounds may indicate ineffective ventilation, decreased perfusion, and fluid accumulation. Stridor a harsh, high-pitched sound primarily heard on inspiration indicates airway obstruction. 39. Nurse Sarah is changing the tapes on a tracheostomy tube. The client coughs and the tube is dislodged. The initial nursing action is to: ANSWER: Grasp the retention sutures to spread the opening 40. A nurse is taking care of a client with Rheumatoid Arthritis and was ordered to collect feces for occult blood exam. Which of the following statements if made by the client need for further instructions? ANSWER: “I may continue taking my Arcoxia 72 hours prior to collection of the sample”. 41. The client being seen in a physician’s office has just been scheduled for a barium swallow the next day. The nurse writes down which of the following instructions for the client to follow before the test? Now is the best time to be the person you dream of becoming P a g e | 16 ANSWER: Fast for 8 hours before the test The patient should maintain NPO status after midnight. Food and fluid in the stomach prevents barium from accurately outlining the GI tract, and the radiographic result may be misleading. 42. A nurse is taking care of client who has just been scheduled for a barium swallow the next day. The nurse must provide the following instructions for the client after the test except: ANSWER: Expect that the bowel movement not to occur within 2 days The evening before the BE, administer cathartics such as magnesium citrate (laxative) or other cathartics designated by institution policy. After the BE study, assess the patient for excavation of the barium. Retained barium may cause a hardened impaction (increase OFI). Stool will be light colored until all barium has expelled. 43. The client has undergone esophagogastroduodenoscopy. The nurse places highest priority on which of the following items as part of the client’s care plan? ANSWER: Assessing for the return of the gag reflex The patient is usually given a preprocedure intravenous (IV) sedative such as midazolam (Versed). The patient pharynx is anesthetized by spraying it with lidocaine hydrochloride (Xylocaine). Therefore do not allow the patient to eat or drink until gag reflex returns (usually about 2 to 4 hours). 44. The nurse determines that the client needs further information if the client makes which of the following statements? ANSWER: “I’m glad I don’t have to lie still for this procedure”. 45. The science of Unitary Human Being and Principles of Homeodynamics are proposed by Martha Rogers. According to Benner’s stages of nursing expertise, a nurse with 2 to 3 years of experience who can coordinate multiple complex nursing care demands is at which stage? ANSWER: Competent Novice - No experience, governed by rules, limited and inflexible, task oriented. Advance Beginner - Demonstrates marginally acceptable performance Competent - has 2-3 yrs. of experience, consciously plans nursing care Proficient - >3-5 yrs. of experience, perceives the situation as a whole rather than parts Expert - Has intuitive grasp of nursing situation. 46. The nurse has given post-procedure instructions to a client who underwent colonoscopy. The nurse determines that the client needs further instructions if the client stated that: ANSWER: It is all right to drive once the client has been home for an hour or so 47. The nurse is caring for a client who is receiving total parenteral nutrition (TPN) via a central line. Which nursing intervention specifically would provide assessment data related to the most common complication related to TPN? ANSWER: Monitoring the temperature – infection Infection is always a concern because the high concentration of dextrose contained in TPN provides excellent medium for bacterial growth. Strict aseptic technique is important while changing bottles containing the TPN solution, tubing, filters, and dressings. Because the catheter is in major blood vessel, any infection would spread rapidly throughout the body. 48. A nurse is preparing to change the TPN solution bag and tubing. The client’s central venous line is located in the right subclavian vein. The nurse asks the client to do who of the following most essential items during the tubing change? ANSWER: Take a deep breath, hold it, and bear down. The client should be asked to perform the Valsalva maneuver during tubing changes. This helps avoid air embolism during tubing changes. The nurse asks the client to take a deep breath, hold it, and bear down. If the IV line is on the right, the client turns his or her head to the left. This position will increase intrathoracic pressure. Options A and C are inappropriate and could cause the potential for an air embolism during the tubing change. 49. A nurse is making initial rounds at the beginning of the shift. The TPN bag of an assigned client is empty. Which of the following solutions readily available on the nursing unit should the nurse hang until another TPN solution is mixed and delivered to the nursing unit? ANSWER: 10% dextrose in water 50. A nurse is inserting a nasogastric tube in an adult client. During the procedure, the client begins to cough and has difficulty of breathing. Which of the following is the most appropriate nursing action? ANSWER: Pull back on the tube and wait until the respiratory distress subsides 51. The nurse checks for residual before administering a bolus tube feeding to a client with nasogastric tube and obtains a residual amount of 150 mL. What is the appropriate action for the nurse to take? ANSWER: Hold the feeding. 52. A nurse is preparing to administer medication through a nasogastric tube that is connected to suction. To administer accurately, the nurse would: Now is the best time to be the person you dream of becoming P a g e | 17 ANSWER: Clamp the nasogastric tube for 30 minutes following administration of the medication 53. A diabetes nurse educator is providing health teaching regarding the proper method of blood glucose determination through skin puncture. The nurse needs to reinforce the teaching if the client identifies which of the following statements? ANSWER: “I should select the central tip of the finger which has more dense blood supply”. 54. What is the most important intervention the nurse can perform to prevent nosocomial infections associated with enteral nutrition? ANSWER: Wearing clean gloves when handling the feeding system 55. The nurse is assessing a stoma prolapse in a client with colostomy. The nurse would observe which of the following if the stoma prolapsed occurred? ANSWER: Protruding stoma A prolapsed stoma is one which the bowel protruded through the stoma. A stoma retraction is characterized by sinking of the stoma. Ischemia of the stoma would be associated with dusky or bluish color . A stoma with a narrowed opening at the level of the skin or fascia is said to be stenosed. 56. Goal Attainment Theory is proposed by Imogene King. A student nurse asked her clinical instructor about Benner’s “Proficient” nurse level. The C.I. did not incorrectly answer the question when she stated that the nurses under this level: ANSWER: Perceives situation as a whole rather than in terms of parts 57. The client with a new colostomy is concerned about the odor from stool in the ostomy drainage bag. The nurse teaches the client to include which of the following foods in the diet to reduce odor? ANSWER: Yogurt The client should be taught to include deodorizing foods in the diet, such a beet greens, parsley, buttermilk, and yogurt. Spinach also reduces odor but is a gas forming food as well. Broccoli, cucumbers, and eggs are gas forming foods. 58. The nurse instructs the ileostomy client to do which of the following as part of essential care of the stoma? ANSWER: Cleanse the peristomal skin meticulously The peristomal skin must receive meticulous cleansing because the ileostomy drainage has more enzymes and is more caustic to the skin than colostomy drainage. Foods such as nuts and those with seeds will pass through the ileostomy. The client should be taught that these foods will remain undigested. The area below the ileostomy may be massaged if needed if the ileostomy becomes blocked by high fiber foods. Fluid intake should be maintained to at least six to eight glasses of water per day to prevent dehydration. 59. The client has just had surgery to create an ileostomy. The nurse assesses the client in the immediate post-operative period for which of the following most frequent complication of this type of surgery? ANSWER: Fluid and electrolyte imbalance 60. A client has an order for “enemas until clear” before major bowel surgery. After preparing the equipment and solution, the nurse assists the client into which of the following positions to administer the enema? ANSWER: Left-lateral Sim’s position Ask the client to lie on the side (preferably the Left-lateral Sim’s position). The colon’s position within the body makes this position the most effective. 61. You are to perform a complete physical assessment to Mr. Mogul Khan, 48 years old, diagnosed with a certain chronic obstructive pulmonary disease. You expect to hear which of the following sounds during percussion of his lung fields? ANSWER: Hyper resonance Flat Sound- Extremely dull (Elicited in Muscles and bones) Dull Sound- Muffled (elicited in liver, spleen and heart) Resonant- Hallow (elicited in a normal lung); Hyperresonant- Booming (elicited in lungs with COPD) Tympany - Drum-like (elicited in Stomach) 62. You were assigned to care for a patient who has suffered from second-degree burns all over his upper extremities just last night. During examination, you noted circumscribed, oval masses, filled with serous fluids that are more than 1 cm. You properly document this as: ANSWER: Bullae Pustule- Circumscribed elevation of skin filled with serous fluid and pus Bullae- thin walled blister greater than .5 cm with serous fluid Now is the best time to be the person you dream of becoming P a g e | 20 82. Which statement should the nurse make when teaching the client about taking oral glucocorticoids? ANSWER: “Take your medication with meals.” 83. The nurse administers neomycin to a client with hepatic cirrhosis to prevent the formation of: ANSWER: Ammonia Neomycin destroy intestinal flora, which breaks down protein and in the process gives off ammonia. Ammonia at this time is poorly detoxified by the liver and can build up to toxic levels. 84. What is the rationale that supports multidrug treatment for clients with tuberculosis? ANSWER: Multiple drugs reduce development of resistant strains of the bacteria Use of a combination of anti-tuberculosis drugs slows the rate at which the organism develops drug resistance. Combination therapy also appears to be more effective than single-drug therapy. Regimens that use only single drugs result in the rapid development of resistance and treatment failure. 85. Jose, a recent graduate of BS Nursing is attending a review class for the November 2014 board exam. The topic of the lecture is Pharmacokinetics. Jose likes to know more about the concept of Pharmacokinetics so he asks the lecturer about the importance of knowing the Pharmacokinetics of a particular drug. The lecturer will mention the following importance of Pharmacokinetics: ANSWER: Pharmacokinetics will explain the details of the chemical interaction between the drug and the target cell, tissue or organ Pharmacokinetics does not discuss the mechanism of action of the drug to the body, instead it describes how the drug moves in the body, and therefore it can help in determining the serum drug concentration 86. The following situations demonstrate an application of a Nurses’ knowledge about the Pharmacodynamics of a given drug, EXCEPT: ANSWER: The Nurse instructs a diabetic patient to avoid rubbing the injection site after SubQ injection of insulin 87. Nurse Kelly noticed that the dose of the opioids given P.O. is higher than that of the I.M. route. Nurse Kelly will be correct in her interpretation by stating that: ANSWER: If the drug is given by the oral route is will be subjected to first pass metabolism. A higher dose of a drug is required if the drug undergoes first pass effect because the drug will be subjected first to metabolism in the liver, therefore inactivating the drug before it reach the systemic circulation 88. If a highly protein bound drugs like Diazepam (98% protein bound), Lorazepam (92% protein bound) or Valproic Acid (92% protein bound) is given to a patient with a liver disease, you will expect: ANSWER: An increased amount of unbound drugs in the blood, therefore more drugs are released into the systemic circulation, increasing its effect 89. Primary level of prevention includes health promotion and illness prevention. Clown Gypsy Group of Companies establishes a physical exercise area in the workplace and encourages all employees to use it. This is an example of which level of prevention? ANSWER: Primary prevention 90. A patient is taking a drug that is known to be toxic to the liver. The patient is being discharged to home. What teaching points related to liver toxicity of the drug that the nurse needs to teach the patient to report to the physician: ANSWER: Body malaise, change in the color of the stool 91. Which of the following will NOT be included in your health teachings when dealing with the excretion of drugs? ANSWER: An acid ash diet will increase the excretion of weak acidic drugs An acid ash diet will acidify the urine therefore promoting the excretion of alkaline drugs like atropine. An alkaline ash diet will alkalinize the urine therefore promoting the excretion of acidic drugs 92. Nurse Michelle is studying different classes of anti-hypertensive, and notes their respective mechanism of action. Which of the following sets of drugs produce an effect of enzyme inhibition to exert their therapeutic action of decreasing the blood pressure? ANSWER: Captopril, Perindopril, Enalapril Losartan, Valsartan and Telmisartan are Angiotensin II receptor antagonist Metoprolol, Atenolol and Propranolol are beta receptor antagonist Methyldopa and Clonidine are Alpha 2 receptor agonists, these drugs do not directly affect enzymes, but reacts with receptors causing or inhibiting a response Captopril, Perindopril and Enalapril are Angiotensin Converting Enzyme Inhibitors that prevents the formation of Angiotensin II that can cause vasoconstriction Now is the best time to be the person you dream of becoming P a g e | 21 93. Nurse Marilyn is taking care of end stage cancer patient. She will be administering Morphine Sulfate and Tramadol for pain. She is aware that it can result to additive interaction, causing greater pain control. She is aware of the other potential advantage of this type of interaction which includes: ANSWER: Lower doses of each drug can be administered, which can decrease the probability of adverse reactions 94. A client confided to you that she experiences cramping abdominal pains and diarrheic episodes upon ingestion of milk and dairy products. She expressed her curiosity regarding lactose intolerance and requested information regarding this condition. All of the following are inappropriate health teaching, aside from: ANSWER: “It’s a condition associated with insufficient lactase, a digestive enzyme.” Lactose intolerance is a condition associated with insufficient or absent lactase, a disaccharidase needed to transform lactose into galactose and glucose. The lack of lactase means the intestines cannot absorb lactose which results in the typical symptoms of LI: diarrhea, abdominal pain, distention, flatulence, nausea etc. Milk with a lower fat content has a higher concentration of lactose. It is not an allergic reaction (milk allergy is a completely separate condition). 95. You have noted that client with a history of cardiovascular diseases was advised to increase his intake of soluble fiber. This dietary modification, in this case, is necessary because: ANSWER: Soluble fiber has been proven to decrease serum cholesterol levels. Soluble fiber is known to decrease serum LDL and cholesterol levels, helpful in clients with CVDs. Choices B and D are health benefits from ingesting insoluble fiber, not soluble. Choice C, although partly true, is not the top priority as the situation did not state that the client has a weight problem. 96. Nurse Arya is conducting a nutritional assessment of Lito, a child who is an inhabitant of a rural village which had been hit with drought and famine. Which of the following assessment findings would most likely lead Nurse Arya to suspect that the Lito has kwashiorkor? ANSWER: Presence of a pot belly The main symptom of kwashiorkor is extensive edema hence a child suffering from this condition would have a puffy appearance and abdominal edema (Choice B). Choices A, C and D are all more commonly associated with marasmus and not kwashiorkor. 97. Mr. Snorlax is a 38-year old client who wants to institute dietary and lifestyle changes in order to decrease his chances of having a cardiac-related event in the future. All of the following must be included in your health teaching, aside from: ANSWER: “A low HDL level means you are reducing your chances of having heart disease.” 98. While conducting a health class on the benefits of eating food with unsaturated fat instead of saturated, you would know that further teaching is unnecessary if the client states: ANSWER: “I should stop using coconut oil when cooking.” Coconut oil, palm oil and chocolate, although coming from plant sources, contain substantially more saturated fat than unsaturated. Choice B is wrong because fish, especially fatty fish, contain Omega-3 fatty acid, a type of polyunsaturated fat that is helpful in decreasing the chances for heart disease. Choice D is also incorrect. Hydrogenation is process that turns unsaturated fat into saturated fat; hence hydrogenated margarine is very high in saturated fat. 99. Mio is a 28-year old married woman who wants to use topical retinoic acid (Avita) to decrease the fine wrinkles on her face, and asks you about what she should know about this medication. Which of the following would be a suitable answer? ANSWER: “We have to ascertain first that you are not pregnant.” ODB: NURSING FACTS AND BELIEFS NURSING THERAPEUTIC 1 Nursing Care/Positioning AFTER the procedure: 1. Thoracentesis- Unaffected side 2. Lobectomy- Unaffected side, with chest tube 3. Segmentectomy- Unaffected side, with chest tube 4. Pneumonectomy- Affected side (slightly affected side) No complete lateral turning to prev. mediastinal shift), No chest tube 5. Eye surgery- Unoperated side down (unaffected side) 6. Liver biopsy- Right side 7. Lumbar puncture- Flat on bed 8. Lower spinal surgery- Flat on bed, log rolling (turning technique) 9. Cervical spinal surgery- Slight elevation of head Now is the best time to be the person you dream of becoming P a g e | 22 10. Cardiac catheterization- Bed rest X 24 hours, affected extremity in Extension, sand bag over site, assess peripheral pulses 11. Hip replacement- Affected limb Abducted 12. Amputation- Elevate extremity for 24 hours 13. Supratentorial surgery- Semi-Fowler’s position 14. Infratentorial surgery- Flat position, avoid neck flexion Nursing care/positioning DURING the procedure: 1. Paracentesis- Sitting position 2. Thoracentesis- Upright leaning on overbed table, sitting Straddling a chair 3. Insertion of TPN catheter- Trendelenburg position 4. Enema Adult- left lateral position Infant/small children- dorsal recumbent 5. TURP- Lithotomy position, cystoclysis 6. Female Catheterization- Dorsal recumbent Important Nursing Care BEFORE THE PROCEDURE 1. Paracentesis- Empty bladder, weigh patient 2. Bowel surgery/colonoscopy- Cleansing Enema 3. Liver Biopsy- Administer Vitamin K to prevent bleeding 4. CT Scan with dye/IVP- Assess allergy 5. MRI- Asses for claustrophobia. Remove metals 6. Ultrasound of KUB-P- Full Bladder 7. Ultrasound of LGBP- NPO FMN, Laxative HS 8. ABG- determination Allen’s Test 9. Giving Digitalis- Assess Apical Pulse (5th L ICS MCL) 10. Giving Narcotics- Assess RR POSITIONING FOR 1. Arterial disorders- Put leg on dependent position 2. Venous disorders- Elevate extremity 3. Increased ICP- Head elevated 15 to 30 degrees & Avoid neck flexion 4. COPD- Upright position NURSING ALERTS! 1. Post Hip Replacement- Avoid Hip Flexion/adduction 2. Pulsating abdominal mass- Avoid abdominal palpation 3. Glaucoma- Avoid Mydriatics (Atropine) 4. DVT. Thrombophlebitis- Avoid massaging legs vigorously 5. Suspected Appendicits- Avoid applying heat, giving laxative, Enema 6. Post spinal cord injury- Avoid flexion of the neck 7. Increased ICP/IOP- Avoid coughing, vomiting, Valsalva CHARACTERISTIC SIGNS/SYMPTOMS AND DISORDERS Meniere’s Disease- Vertigo, tinnitus and hearing loss Retinal detachment- Flashes, floaters and veils Glaucoma- Increased IOP, loss of peripheral vision, tunnel vision and Halos, rainbows around lights Cataract- Opacity of the lens, painless loss of vision Parkinson’s disease- Bradykinesia, cogwheel rigidity, shuffling propulsive gait and Pill rolling tremor Guillain Barre syndrome- Ascending paralysis Myasthenia Graves- Ptosis, dysphagia, respiratory paralysis Multiple Sclerosis- Charcot’s Triad (scanning speech, intention tremors, nystagmus) Poliomyelitis- Flaccid paralysis Fracture- Crepitus Abdominal aortic aneurysm- Pulsating abdominal mass Gouty arthritis- Tophi Rheumatoid arthritis- Subcutaneous nodules, morning stiffness Osteoarthritis- Heberdens nodules, Bouchard nodules Acromegaly- Coarse facial features, wide hands and feet Cretinism- Physical and mental retardation Graves disease- Exopthalmos, tachycardia, heat intolerance Syndrome of inappropriate antidiuretic hormone secretion (SIADH)- Water intoxication, hyponatremia Diabetes insipidus- Polyuria, decreased specific gravity of urine Now is the best time to be the person you dream of becoming P a g e | 25 Reposition a bedridden client at least every 2 hours and achair bound client every hour Common fecal elimination problem includes constipation, diarrhea, incontinence, flatulence Lack of exercise, irregular defecation habits, bland diet and overuse of laxative are all thought to contribute to constipation Sufficient fluid and fiber intake are required to keep feces soft An adverse effect of constipation is straining during defecation An adverse effect of prolonged diarrhea is fluid and electrolyte imbalances When inspecting the client stool, the nurse must observe its color, consistency, shape, amount, odor, and the constituents Digital removal of an impaction should be carried out gently Enema is used to relieve constipation, fecal impaction, flatulence and is also used for evacuation and in lowering body temperature Proper positioning must be observed in doing enema. Left lateral position for adults and dorsal recumbent position for child and infants Urinary elimination depends on normal functioning of the urinary, cardiovascular and nervous system Alteration in urine production and elimination includes polyuria, oliguria, anuria, frequency, nocturia, urgency, enuresis, hematuria, incontinence and retention Incontinence can be physically and emotionally distressing to client Clients with urinary retention is at risk for tract infection The most common cause of urinary tract infection is invasive procedure Goals for client with problems with urinary elimination problems includes maintaining or restoring normal elimination patterns and preventing skin breakdown Urinary catheterization is frequently required for clients with urinary retention but is only performed when all other measures to facilitate voiding fails Gradual decompression should be done in doing straight catheterization Fr 16 – 18 is used from male client and Fr 12 – 14 is used for female client for catheterization For retention catheter, inflate the balloon with 5 ml of sterile NSS For client with retention catheter, acidifying urine is a must. Food such as meat, fish, eggs and cereals Normal pH of urine is 6 or a range of 4.6 – 8 Clean voided specimen is used for routine urinalysis Midstream urine specimen is used for urine culture Timed urine specimen collection is used to assess the ability of the kidney to concentrate and dilute urine; determine level of specific constituents; determine disorders of glucose metabolism If the client or staff forgets and discard the client’s urine during times collection, the procedure must be restarted from the beginning To collect a stool specimen for infant, the stool is scraped from the diaper For occult blood examination, the client should be instructed to avoid dark-colored food, red meat, iron andn hemoglobin rich food for 48 – 74 hours Avoid collecting specimen during menstrual period Sputum specimen is best collected early morning to help the client expectorate all the secretions that has been accumulated at night Method of sputum collection includes deep breathing and coughing exercise, suctioning and chest physiotherapy Blood test are the most commonly used diagnostic test and can provide valuable information about the hematologic system and other body system Walker promotes more stability compare to cane and crutches Crutches observes different gaits such as: 4 point gait, 3 point gait and 2 point gait, plus swing to and swing through Cane should always be places on the stronger side of the body Client should always be free from restraint Reason for placing the client on a restraint includes: disruptive behavior, procedure and transfer Never anchor the restraint on the side rail Free movement should be provided when placing the client on a restraint Two types of visualization are direct and indirect. Direct methods make use of gadget or instrument to visualize a body area/organ. Indirect methods make use of dye, electrical impulses KUB X-ray (kidney, ureters, bladders) are painless For voiding cystourethrogram films are taken before, during and after voiding Retrograde pyelogram (RPG) outlines the pelvis and ureters. Epinephrine at the bedside should be prepared A pink tinged urine is normal after cystoscopy because of the irritation of the mucus membrane Now is the best time to be the person you dream of becoming P a g e | 26 Distending bladder for KUB ultrasound is done for better imaging A 24 hours bed rest after renal biopsy is encouraged to prevent bleeding Blood Transfusion is used to improve the oxygen carrying capacity of the blood and restore the blood volume 0.9% NaCl is the only fluid solution allowed for blood transfusion Label the blood and warm the blood at a room temperature Use needle gauge 18 or 19 for BT to allow easy flow of the blood Do not mix medication with blood transfusion Stop the transfusion immediately for any complication that might occur during blood transfusion Fundamentals of Nursing Notes 1 1. A blood pressure cuff that’s too narrow can cause a falsely elevated blood pressure reading. 2. When preparing a single injection for a patient who takes regular and neutral protein Hagedorn insulin, the nurse should draw the regular insulin into the syringe first so that it does not contaminate the regular insulin. 3. Rhonchi are the rumbling sounds heard on lung auscultation. They are more pronounced during expiration than during inspiration. 4. Gavage is forced feeding, usually through a gastric tube (a tube passed into the stomach through the mouth). 5. According to Maslow’s hierarchy of needs, physiologic needs (air, water, food, shelter, sex, activity, and comfort) have the highest priority. 6. The safest and surest way to verify a patient’s identity is to check the identification band on his wrist. 7. In the therapeutic environment, the patient’s safety is the primary concern. 8. Fluid oscillation in the tubing of a chest drainage system indicates that the system is working properly. 9. The nurse should place a patient who has a Sengstaken-Blakemore tube in semi-Fowler position. 10. The nurse can elicit Trousseau’s sign by occluding the brachial or radial artery. Hand and finger spasms that occur during occlusion indicate Trousseau’s sign and suggest hypocalcemia. 11. For blood transfusion in an adult, the appropriate needle size is 16 to 20G. 12. Intractable pain is pain that incapacitates a patient and can’t be relieved by drugs. 13. In an emergency, consent for treatment can be obtained by fax, telephone, or other telegraphic means. 14. Decibel is the unit of measurement of sound. 15. Informed consent is required for any invasive procedure. 16. A patient who can’t write his name to give consent for treatment must make an X in the presence of two witnesses, such as a nurse, priest, or physician. 17. The Z-track I.M. injection technique seals the drug deep into the muscle, thereby minimizing skin irritation and staining. It requires a needle that’s 1″ (2.5 cm) or longer. 18. In the event of fire, the acronym most often used is RACE. (R) Remove the patient. (A) Activate the alarm. (C) Attempt to contain the fire by closing the door. (E) Extinguish the fire if it can be done safely. 19. A registered nurse should assign a licensed vocational nurse or licensed practical nurse to perform bedside care, such as suctioning and drug administration. 20. If a patient can’t void, the first nursing action should be bladder palpation to assess for bladder distention. 21. The patient who uses a cane should carry it on the unaffected side and advance it at the same time as the affected extremity. 22. To fit a supine patient for crutches, the nurse should measure from the axilla to the sole and add 2″ (5 cm) to that measurement. 23. Assessment begins with the nurse’s first encounter with the patient and continues throughout the patient’s stay. The nurse obtains assessment data through the health history, physical examination, and review of diagnostic studies. 24. The appropriate needle size for insulin injection is 25G and 5/8″ long. 25. Residual urine is urine that remains in the bladder after voiding. The amount of residual urine is normally 50 to 100 ml. 26. The five stages of the nursing process are assessment, nursing diagnosis, planning, implementation, and evaluation. 27. Assessment is the stage of the nursing process in which the nurse continuously collects data to identify a patient’s actual and potential health needs. 28. Nursing diagnosis is the stage of the nursing process in which the nurse makes a clinical judgment about individual, family, or community responses to actual or potential health problems or life processes. 29. Planning is the stage of the nursing process in which the nurse assigns priorities to nursing diagnoses, defines short- term and long-term goals and expected outcomes, and establishes the nursing care plan. Now is the best time to be the person you dream of becoming P a g e | 27 30. Implementation is the stage of the nursing process in which the nurse puts the nursing care plan into action, delegates specific nursing interventions to members of the nursing team, and charts patient responses to nursing interventions. 31. Evaluation is the stage of the nursing process in which the nurse compares objective and subjective data with the outcome criteria and, if needed, modifies the nursing care plan. 32. Before administering any “as needed” pain medication, the nurse should ask the patient to indicate the location of the pain. 33. Jehovah’s Witnesses believe that they shouldn’t receive blood components donated by other people. 34. To test visual acuity, the nurse should ask the patient to cover each eye separately and to read the eye chart with glasses and without, as appropriate. 35. When providing oral care for an unconscious patient, to minimize the risk of aspiration, the nurse should position the patient on the side. 36. During assessment of distance vision, the patient should stand 20′ (6.1 m) from the chart. 37. For a geriatric patient or one who is extremely ill, the ideal room temperature is 66° to 76° F (18.8° to 24.4° C). 38. Normal room humidity is 30% to 60%. 39. Hand washing is the single best method of limiting the spread of microorganisms. Once gloves are removed after routine contact with a patient, hands should be washed for 10 to 15 seconds. 40. To perform catheterization, the nurse should place a woman in the dorsal recumbent position. 41. A positive Homan’s sign may indicate thrombophlebitis. 42. Electrolytes in a solution are measured in milliequivalents per liter (mEq/L). A milliequivalent is the number of milligrams per 100 milliliters of a solution. 43. Metabolism occurs in two phases: anabolism (the constructive phase) and catabolism (the destructive phase). 44. The basal metabolic rate is the amount of energy needed to maintain essential body functions. It’s measured when the patient is awake and resting, hasn’t eaten for 14 to 18 hours, and is in a comfortable, warm environment. 45. The basal metabolic rate is expressed in calories consumed per hour per kilogram of body weight. 46. Dietary fiber (roughage), which is derived from cellulose, supplies bulk, maintains intestinal motility, and helps to establish regular bowel habits. 47. Alcohol is metabolized primarily in the liver. Smaller amounts are metabolized by the kidneys and lungs. 48. Petechiae are tiny, round, purplish red spots that appear on the skin and mucous membranes as a result of intradermal or submucosal hemorrhage. 49. Purpura is a purple discoloration of the skin that’s caused by blood extravasation. 50. According to the standard precautions recommended by the Centers for Disease Control and Prevention, the nurse shouldn’t recap needles after use. Most needle sticks result from missed needle recapping. 51. The nurse administers a drug by I.V. push by using a needle and syringe to deliver the dose directly into a vein, I.V. tubing, or a catheter. 52. When changing the ties on a tracheostomy tube, the nurse should leave the old ties in place until the new ones are applied. 53. A nurse should have assistance when changing the ties on a tracheostomy tube. 54. A filter is always used for blood transfusions. 55. A four-point (quad) cane is indicated when a patient needs more stability than a regular cane can provide. 56. A good way to begin a patient interview is to ask, “What made you seek medical help?” 57. When caring for any patient, the nurse should follow standard precautions for handling blood and body fluids. 58. Potassium (K+) is the most abundant cation in intracellular fluid. 59. In the four-point, or alternating, gait, the patient first moves the right crutch followed by the left foot and then the left crutch followed by the right foot. 60. In the three-point gait, the patient moves two crutches and the affected leg simultaneously and then moves the unaffected leg. 61. In the two-point gait, the patient moves the right leg and the left crutch simultaneously and then moves the left leg and the right crutch simultaneously. 62. The vitamin B complex, the water-soluble vitamins that are essential for metabolism, include thiamine (B1), riboflavin (B2), niacin (B3), pyridoxine (B6), and cyanocobalamin (B12). 63. When being weighed, an adult patient should be lightly dressed and shoeless. 64. Before taking an adult’s temperature orally, the nurse should ensure that the patient hasn’t smoked or consumed hot or cold substances in the previous 15 minutes. Now is the best time to be the person you dream of becoming P a g e | 30 131. When preparing for a skull X-ray, the patient should remove all jewelry and dentures. 132. The fight-or-flight response is a sympathetic nervous system response. 133. Bronchovesicular breath sounds in peripheral lung fields are abnormal and suggest pneumonia. 134. Wheezing is an abnormal, high-pitched breath sound that’s accentuated on expiration. 135. Wax or a foreign body in the ear should be flushed out gently by irrigation with warm saline solution. 136. If a patient complains that his hearing aid is “not working,” the nurse should check the switch first to see if it’s turned on and then check the batteries. 137. The nurse should grade hyperactive biceps and triceps reflexes as +4. 138. If two eye medications are prescribed for twice-daily instillation, they should be administered 5 minutes apart. 139. In a postoperative patient, forcing fluids helps prevent constipation. 140. A nurse must provide care in accordance with standards of care established by the American Nurses Association, state regulations, and facility policy. 141. The kilocalorie (kcal) is a unit of energy measurement that represents the amount of heat needed to raise the temperature of 1 kilogram of water 1° C. 142. As nutrients move through the body, they undergo ingestion, digestion, absorption, transport, cell metabolism, and excretion. 143. The body metabolizes alcohol at a fixed rate, regardless of serum concentration. 144. In an alcoholic beverage, proof reflects the percentage of alcohol multiplied by 2. For example, a 100-proof beverage contains 50% alcohol. 145. A living will is a witnessed document that states a patient’s desire for certain types of care and treatment. These decisions are based on the patient’s wishes and views on quality of life. 146. The nurse should flush a peripheral heparin lock every 8 hours (if it wasn’t used during the previous 8 hours) and as needed with normal saline solution to maintain patency. 147. Quality assurance is a method of determining whether nursing actions and practices meet established standards. 148. The five rights of medication administration are the right patient, right drug, right dose, right route of administration, and right time. 149. The evaluation phase of the nursing process is to determine whether nursing interventions have enabled the patient to meet the desired goals. 150. Outside of the hospital setting, only the sublingual and translingual forms of nitroglycerin should be used to relieve acute anginal attacks. 151. The implementation phase of the nursing process involves recording the patient’s response to the nursing plan, putting the nursing plan into action, delegating specific nursing interventions, and coordinating the patient’s activities. 152. The Patient’s Bill of Rights offers patients guidance and protection by stating the responsibilities of the hospital and its staff toward patients and their families during hospitalization. 153. To minimize omission and distortion of facts, the nurse should record information as soon as it’s gathered. 154. When assessing a patient’s health history, the nurse should record the current illness chronologically, beginning with the onset of the problem and continuing to the present. 155. When assessing a patient’s health history, the nurse should record the current illness chronologically, beginning with the onset of the problem and continuing to the present. 156. A nurse shouldn’t give false assurance to a patient. 157. After receiving preoperative medication, a patient isn’t competent to sign an informed consent form. 158. When lifting a patient, a nurse uses the weight of her body instead of the strength in her arms. 159. A nurse may clarify a physician’s explanation about an operation or a procedure to a patient, but must refer questions about informed consent to the physician. 160. When obtaining a health history from an acutely ill or agitated patient, the nurse should limit questions to those that provide necessary information. 161. If a chest drainage system line is broken or interrupted, the nurse should clamp the tube immediately. 162. The nurse shouldn’t use her thumb to take a patient’s pulse rate because the thumb has a pulse that may be confused with the patient’s pulse. 163. An inspiration and an expiration count as one respiration. 164. Eupnea is normal respiration. 165. During blood pressure measurement, the patient should rest the arm against a surface. Using muscle strength to hold up the arm may raise the blood pressure. Now is the best time to be the person you dream of becoming P a g e | 31 166. Major, unalterable risk factors for coronary artery disease include heredity, sex, race, and age. 167. Inspection is the most frequently used assessment technique. 168. Family members of an elderly person in a long-term care facility should transfer some personal items (such as photographs, a favorite chair, and knickknacks) to the person’s room to provide a comfortable atmosphere. 169. Pulsus alternans is a regular pulse rhythm with alternating weak and strong beats. It occurs in ventricular enlargement because the stroke volume varies with each heartbeat. 170. The upper respiratory tract warms and humidifies inspired air and plays a role in taste, smell, and mastication. 171. Signs of accessory muscle use include shoulder elevation, intercostal muscle retraction, and scalene and sternocleidomastoid muscle use during respiration. 172. When patients use axillary crutches, their palms should bear the brunt of the weight. 173. Activities of daily living include eating, bathing, dressing, grooming, toileting, and interacting socially. 174. Normal gait has two phases: the stance phase, in which the patient’s foot rests on the ground, and the swing phase, in which the patient’s foot moves forward. 175. The phases of mitosis are prophase, metaphase, anaphase, and telophase. 176. The nurse should follow standard precautions in the routine care of all patients. 177. The nurse should use the bell of the stethoscope to listen for venous hums and cardiac murmurs. 178. The nurse can assess a patient’s general knowledge by asking questions such as “Who is the president of the United States?” 179. Cold packs are applied for the first 20 to 48 hours after an injury; then heat is applied. During cold application, the pack is applied for 20 minutes and then removed for 10 to 15 minutes to prevent reflex dilation (rebound phenomenon) and frostbite injury. 180. The pons is located above the medulla and consists of white matter (sensory and motor tracts) and gray matter (reflex centers). 181. The autonomic nervous system controls the smooth muscles. 182. A correctly written patient goal expresses the desired patient behavior, criteria for measurement, time frame for achievement, and conditions under which the behavior will occur. It’s developed in collaboration with the patient. 183. Percussion causes five basic notes: tympany (loud intensity, as heard over a gastric air bubble or puffed out cheek), hyperresonance (very loud, as heard over an emphysematous lung), resonance (loud, as heard over a normal lung), dullness (medium intensity, as heard over the liver or other solid organ), and flatness (soft, as heard over the thigh). 184. The optic disk is yellowish pink and circular, with a distinct border. 185. A primary disability is caused by a pathologic process. A secondary disability is caused by inactivity. 186. Nurses are commonly held liable for failing to keep an accurate count of sponges and other devices during surgery. 187. The best dietary sources of vitamin B6 are liver, kidney, pork, soybeans, corn, and whole-grain cereals. 188. Iron-rich foods, such as organ meats, nuts, legumes, dried fruit, green leafy vegetables, eggs, and whole grains, commonly have a low water content. 189. Collaboration is joint communication and decision making between nurses and physicians. It’s designed to meet patients’ needs by integrating the care regimens of both professions into one comprehensive approach. 190. Bradycardia is a heart rate of fewer than 60 beats/minute. 191. A nursing diagnosis is a statement of a patient’s actual or potential health problem that can be resolved, diminished, or otherwise changed by nursing interventions. 192. During the assessment phase of the nursing process, the nurse collects and analyzes three types of data: health history, physical examination, and laboratory and diagnostic test data. 193. The patient’s health history consists primarily of subjective data, information that’s supplied by the patient. 194. The physical examination includes objective data obtained by inspection, palpation, percussion, and auscultation. 195. When documenting patient care, the nurse should write legibly, use only standard abbreviations, and sign each entry. The nurse should never destroy or attempt to obliterate documentation or leave vacant lines. 196. Factors that affect body temperature include time of day, age, physical activity, phase of menstrual cycle, and pregnancy. 197. The most accessible and commonly used artery for measuring a patient’s pulse rate is the radial artery. To take the pulse rate, the artery is compressed against the radius. 198. In a resting adult, the normal pulse rate is 60 to 100 beats/minute. The rate is slightly faster in women than in men and much faster in children than in adults. 199. Laboratory test results are an objective form of assessment data. Now is the best time to be the person you dream of becoming P a g e | 32 200. The measurement systems most commonly used in clinical practice are the metric system, apothecaries’ system, and household system. 201. Before signing an informed consent form, the patient should know whether other treatment options are available and should understand what will occur during the preoperative, intraoperative, and postoperative phases; the risks involved; and the possible complications. The patient should also have a general idea of the time required from surgery to recovery. In addition, he should have an opportunity to ask questions. 202. A patient must sign a separate informed consent form for each procedure. 203. During percussion, the nurse uses quick, sharp tapping of the fingers or hands against body surfaces to produce sounds. This procedure is done to determine the size, shape, position, and density of underlying organs and tissues; elicit tenderness; or assess reflexes. 204. Ballottement is a form of light palpation involving gentle, repetitive bouncing of tissues against the hand and feeling their rebound. 205. A foot cradle keeps bed linen off the patient’s feet to prevent skin irritation and breakdown, especially in a patient who has peripheral vascular disease or neuropathy. 206. Gastric lavage is flushing of the stomach and removal of ingested substances through a nasogastric tube. It’s used to treat poisoning or drug overdose. 207. During the evaluation step of the nursing process, the nurse assesses the patient’s response to therapy. 208. Bruits commonly indicate life- or limb-threatening vascular disease. 209. O.U. means each eye. O.D. is the right eye, and O.S. is the left eye. 210. To remove a patient’s artificial eye, the nurse depresses the lower lid. 211. The nurse should use a warm saline solution to clean an artificial eye. 212. A thready pulse is very fine and scarcely perceptible. 213. Axillary temperature is usually 1° F lower than oral temperature. 214. After suctioning a tracheostomy tube, the nurse must document the color, amount, consistency, and odor of secretions. 215. On a drug prescription, the abbreviation p.c. means that the drug should be administered after meals. 216. After bladder irrigation, the nurse should document the amount, color, and clarity of the urine and the presence of clots or sediment. 217. After bladder irrigation, the nurse should document the amount, color, and clarity of the urine and the presence of clots or sediment. 218. Laws regarding patient self-determination vary from state to state. Therefore, the nurse must be familiar with the laws of the state in which she works. 219. Gauge is the inside diameter of a needle: the smaller the gauge, the larger the diameter. 220. An adult normally has 32 permanent teeth. Fundamentals of Nursing Notes 2 1. After turning a patient, the nurse should document the position used, the time that the patient was turned, and the findings of skin assessment. 2. PERRLA is an abbreviation for normal pupil assessment findings: pupils equal, round, and reactive to light with accommodation. 3. When percussing a patient’s chest for postural drainage, the nurse’s hands should be cupped. 4. When measuring a patient’s pulse, the nurse should assess its rate, rhythm, quality, and strength. 5. Before transferring a patient from a bed to a wheelchair, the nurse should push the wheelchair footrests to the sides and lock its wheels. 6. When assessing respirations, the nurse should document their rate, rhythm, depth, and quality. 7. For a subcutaneous injection, the nurse should use a 5/8″ to 1″ 25G needle. 8. The notation “AA & O × 3” indicates that the patient is awake, alert, and oriented to person (knows who he is), place (knows where he is), and time (knows the date and time). 9. Fluid intake includes all fluids taken by mouth, including foods that are liquid at room temperature, such as gelatin, custard, and ice cream; I.V. fluids; and fluids administered in feeding tubes. Fluid output includes urine, vomitus, and drainage (such as from a nasogastric tube or from a wound) as well as blood loss, diarrhea or feces, and perspiration. 10. After administering an intradermal injection, the nurse shouldn’t massage the area because massage can irritate the site and interfere with results. Now is the best time to be the person you dream of becoming P a g e | 35 81. An ascending colostomy drains fluid feces. A descending colostomy drains solid fecal matter. 82. A folded towel (scrotal bridge) can provide scrotal support for the patient with scrotal edema caused by vasectomy, epididymitis, or orchitis. 83. When giving an injection to a patient who has a bleeding disorder, the nurse should use a small-gauge needle and apply pressure to the site for 5 minutes after the injection. 84. Platelets are the smallest and most fragile formed element of the blood and are essential for coagulation. 85. To insert a nasogastric tube, the nurse instructs the patient to tilt the head back slightly and then inserts the tube. When the nurse feels the tube curving at the pharynx, the nurse should tell the patient to tilt the head forward to close the trachea and open the esophagus by swallowing. (Sips of water can facilitate this action.) 86. Families with loved ones in intensive care units report that their four most important needs are to have their questions answered honestly, to be assured that the best possible care is being provided, to know the patient’s prognosis, and to feel that there is hope of recovery. 87. Double-bind communication occurs when the verbal message contradicts the nonverbal message and the receiver is unsure of which message to respond to. 88. A nonjudgmental attitude displayed by a nurse shows that she neither approves nor disapproves of the patient. 89. Target symptoms are those that the patient finds most distressing. 90. A patient should be advised to take aspirin on an empty stomach, with a full glass of water, and should avoid acidic foods such as coffee, citrus fruits, and cola. 91. For every patient problem, there is a nursing diagnosis; for every nursing diagnosis, there is a goal; and for every goal, there are interventions designed to make the goal a reality. The keys to answering examination questions correctly are identifying the problem presented, formulating a goal for the problem, and selecting the intervention from the choices provided that will enable the patient to reach that goal. 92. Fidelity means loyalty and can be shown as a commitment to the profession of nursing and to the patient. 93. Administering an I.M. injection against the patient’s will and without legal authority is battery. 94. An example of a third-party payer is an insurance company. 95. The formula for calculating the drops per minute for an I.V. infusion is as follows: (volume to be infused × drip factor) ÷ time in minutes = drops/minute 96. On-call medication should be given within 5 minutes of the call. 97. Usually, the best method to determine a patient’s cultural or spiritual needs is to ask him. 98. An incident report or unusual occurrence report isn’t part of a patient’s record, but is an in-house document that’s used for the purpose of correcting the problem. 99. Critical pathways are a multidisciplinary guideline for patient care. 100. When prioritizing nursing diagnoses, the following hierarchy should be used: Problems associated with the airway, those concerning breathing, and those related to circulation. 101. The two nursing diagnoses that have the highest priority that the nurse can assign are Ineffective airway clearance and Ineffective breathing pattern. 102. A subjective sign that a sitz bath has been effective is the patient’s expression of decreased pain or discomfort. 103. For the nursing diagnosis Deficient diversional activity to be valid, the patient must state that he’s “bored,” that he has “nothing to do,” or words to that effect. 104. The most appropriate nursing diagnosis for an individual who doesn’t speak English is Impaired verbal communication related to inability to speak dominant language (English). 105. The family of a patient who has been diagnosed as hearing impaired should be instructed to face the individual when they speak to him. 106. Before instilling medication into the ear of a patient who is up to age 3, the nurse should pull the pinna down and back to straighten the eustachian tube. 107. To prevent injury to the cornea when administering eyedrops, the nurse should waste the first drop and instill the drug in the lower conjunctival sac. 108. After administering eye ointment, the nurse should twist the medication tube to detach the ointment. 109. When the nurse removes gloves and a mask, she should remove the gloves first. They are soiled and are likely to contain pathogens. 110. Crutches should be placed 6″ (15.2 cm) in front of the patient and 6″ to the side to form a tripod arrangement. 111. Listening is the most effective communication technique. 112. Before teaching any procedure to a patient, the nurse must assess the patient’s current knowledge and willingness to learn. Now is the best time to be the person you dream of becoming P a g e | 36 113. Process recording is a method of evaluating one’s communication effectiveness. 114. When feeding an elderly patient, the nurse should limit high-carbohydrate foods because of the risk of glucose intolerance. 115. When feeding an elderly patient, essential foods should be given first. 116. Passive range of motion maintains joint mobility. Resistive exercises increase muscle mass. 117. Isometric exercises are performed on an extremity that’s in a cast. 118. A back rub is an example of the gate-control theory of pain. 119. Anything that’s located below the waist is considered unsterile; a sterile field becomes unsterile when it comes in contact with any unsterile item; a sterile field must be monitored continuously; and a border of 1″ (2.5 cm) around a sterile field is considered unsterile. 120. A “shift to the left” is evident when the number of immature cells (bands) in the blood increases to fight an infection. ADVERTISEMENTS 121. A “shift to the right” is evident when the number of mature cells in the blood increases, as seen in advanced liver disease and pernicious anemia. 122. Before administering preoperative medication, the nurse should ensure that an informed consent form has been signed and attached to the patient’s record. 123. A nurse should spend no more than 30 minutes per 8-hour shift providing care to a patient who has a radiation implant. 124. A nurse shouldn’t be assigned to care for more than one patient who has a radiation implant. 125. Long-handled forceps and a lead-lined container should be available in the room of a patient who has a radiation implant. 126. Usually, patients who have the same infection and are in strict isolation can share a room. 127. Diseases that require strict isolation include chickenpox, diphtheria, and viral hemorrhagic fevers such as Marburg disease. 128. For the patient who abides by Jewish custom, milk and meat shouldn’t be served at the same meal. 129. Whether the patient can perform a procedure (psychomotor domain of learning) is a better indicator of the effectiveness of patient teaching than whether the patient can simply state the steps involved in the procedure (cognitive domain of learning). 130. According to Erik Erikson, developmental stages are trust versus mistrust (birth to 18 months), autonomy versus shame and doubt (18 months to age 3), initiative versus guilt (ages 3 to 5), industry versus inferiority (ages 5 to 12), identity versus identity diffusion (ages 12 to 18), intimacy versus isolation (ages 18 to 25), generativity versus stagnation (ages 25 to 60), and ego integrity versus despair (older than age 60). 131. When communicating with a hearing impaired patient, the nurse should face him. 132. An appropriate nursing intervention for the spouse of a patient who has a serious incapacitating disease is to help him to mobilize a support system. 133. Hyperpyrexia is extreme elevation in temperature above 106° F (41.1° C). 134. Milk is high in sodium and low in iron. 135. When a patient expresses concern about a health-related issue, before addressing the concern, the nurse should assess the patient’s level of knowledge. 136. The most effective way to reduce a fever is to administer an antipyretic, which lowers the temperature set point. 137. When a patient is ill, it’s essential for the members of his family to maintain communication about his health needs. 138. Ethnocentrism is the universal belief that one’s way of life is superior to others. 139. When a nurse is communicating with a patient through an interpreter, the nurse should speak to the patient and the interpreter. 140. In accordance with the “hot-cold” system used by some Mexicans, Puerto Ricans, and other Hispanic and Latino groups, most foods, beverages, herbs, and drugs are described as “cold.” 141. Prejudice is a hostile attitude toward individuals of a particular group. 142. Discrimination is preferential treatment of individuals of a particular group. It’s usually discussed in a negative sense. 143. Increased gastric motility interferes with the absorption of oral drugs. 144. The three phases of the therapeutic relationship are orientation, working, and termination. 145. Patients often exhibit resistive and challenging behaviors in the orientation phase of the therapeutic relationship. 146. Abdominal assessment is performed in the following order: inspection, auscultation, percussion & palpation. 147. When measuring blood pressure in a neonate, the nurse should select a cuff that’s no less than one-half and no more than two-thirds the length of the extremity that’s used. 148. When administering a drug by Z-track, the nurse shouldn’t use the same needle that was used to draw the drug into the syringe because doing so could stain the skin. Now is the best time to be the person you dream of becoming P a g e | 37 149. Sites for intradermal injection include the inner arm, the upper chest, and on the back, under the scapula. 150. When evaluating whether an answer on an examination is correct, the nurse should consider whether the action that’s described promotes autonomy (independence), safety, self-esteem, and a sense of belonging. 151. When answering a question on the NCLEX examination, the student should consider the cue (the stimulus for a thought) and the inference (the thought) to determine whether the inference is correct. When in doubt, the nurse should select an answer that indicates the need for further information to eliminate ambiguity. For example, the patient complains of chest pain (the stimulus for the thought) and the nurse infers that the patient is having cardiac pain (the thought). In this case, the nurse hasn’t confirmed whether the pain is cardiac. It would be more appropriate to make further assessments. 152. Veracity is truth and is an essential component of a therapeutic relationship between a health care provider and his patient. 153. Beneficence is the duty to do no harm and the duty to do good. There’s an obligation in patient care to do no harm and an equal obligation to assist the patient. 154. Nonmaleficence is the duty to do no harm. 155. Frye’s ABCDE cascade provides a framework for prioritizing care by identifying the most important treatment concerns. 156. A = Airway. This category includes everything that affects a patent airway, including a foreign object, fluid from an upper respiratory infection, and edema from trauma or an allergic reaction. 157. B = Breathing. This category includes everything that affects the breathing pattern, including hyperventilation or hypoventilation and abnormal breathing patterns, such as Korsakoff’s, Biot’s, or Cheyne-Stokes respiration. 158. C = Circulation. This category includes everything that affects the circulation, including fluid and electrolyte disturbances and disease processes that affect cardiac output. 159. D = Disease processes. If the patient has no problem with the airway, breathing, or circulation, then the nurse should evaluate the disease processes, giving priority to the disease process that poses the greatest immediate risk. For example, if a patient has terminal cancer and hypoglycemia, hypoglycemia is a more immediate concern. 160. E = Everything else. This category includes such issues as writing an incident report and completing the patient chart. When evaluating needs, this category is never the highest priority. 161. When answering a question on an NCLEX examination, the basic rule is “assess before action.” The student should evaluate each possible answer carefully. Usually, several answers reflect the implementation phase of nursing and one or two reflect the assessment phase. In this case, the best choice is an assessment response unless a specific course of action is clearly indicated. 162. Rule utilitarianism is known as the “greatest good for the greatest number of people” theory. 163. Egalitarian theory emphasizes that equal access to goods and services must be provided to the less fortunate by an affluent society. 164. Active euthanasia is actively helping a person to die. 165. Brain death is irreversible cessation of all brain function. 166. Passive euthanasia is stopping the therapy that’s sustaining life. 167. A third-party payer is an insurance company. 168. Utilization review is performed to determine whether the care provided to a patient was appropriate and cost-effective. 169. A value cohort is a group of people who experienced an out-of-the-ordinary event that shaped their values. 170. Voluntary euthanasia is actively helping a patient to die at the patient’s request. 171. Bananas, citrus fruits, and potatoes are good sources of potassium. 172. Good sources of magnesium include fish, nuts, and grains. 173. Beef, oysters, shrimp, scallops, spinach, beets, and greens are good sources of iron. 174. Intrathecal injection is administering a drug through the spine. 175. When a patient asks a question or makes a statement that’s emotionally charged, the nurse should respond to the emotion behind the statement or question rather than to what’s being said or asked. 176. The steps of the trajectory-nursing model are as follows: 177. Step 1: Identifying the trajectory phase 178. Step 2: Identifying the problems and establishing goals 179. Step 3: Establishing a plan to meet the goals 180. Step 4: Identifying factors that facilitate or hinder attainment of the goals 181. Step 5: Implementing interventions 182. Step 6: Evaluating the effectiveness of the interventions 183. A Hindu patient is likely to request a vegetarian diet. Now is the best time to be the person you dream of becoming P a g e | 40 When assessing the peripheral visual field, the client can see objects in the periphery when looking straight ahead. When testing for the Extraocular Muscle, both eyes of the client coordinately moved in unison with parallel alignment. The client was able to read the newsprint held at a distance of 14 inches. Ears and Hearing Ears: The Auricles are symmetrical and has the same color with his facial skin. The auricles are aligned with the outer canthus of eye. When palpating for the texture, the auricles are mobile, firm and not tender. The pinna recoils when folded. During the assessment of Watch tick test, the client was able to hear ticking in both ears. Nose and Sinus Nose: The nose appeared symmetric, straight and uniform in color. There was no presence of discharge or flaring. When lightly palpated, there were no tenderness and lesions Mouth The lips of the client are uniformly pink; moist, symmetric and have a smooth texture. The client was able to purse his lips when asked to whistle. Teeth and Gums: There are no discoloration of the enamels, no retraction of gums, pinkish in color of gums The buccal mucosa of the client appeared as uniformly pink; moist, soft, glistening and with elastic texture. The tongue of the client is centrally positioned. It is pink in color, moist and slightly rough. There is a presence of thin whitish coating. The smooth palates are light pink and smooth while the hard palate has a more irregular texture. The uvula of the client is positioned in the midline of the soft palate. Neck The neck muscles are equal in size. The client showed coordinated, smooth head movement with no discomfort. The lymph nodes of the client are not palpable. The trachea is placed in the midline of the neck. The thyroid gland is not visible on inspection and the glands ascend during swallowing but are not visible. Thorax, Lungs, and Abdomen Lungs / Chest: The chest wall is intact with no tenderness and masses. There’s a full and symmetric expansion and the thumbs separate 2-3 cm during deep inspiration when assessing for the respiratory excursion. The client manifested quiet, rhythmic and effortless respirations. The spine is vertically aligned. The right and left shoulders and hips are of the same height. Heart: There were no visible pulsations on the aortic and pulmonic areas. There is no presence of heaves or lifts. Abdomen: The abdomen of the client has an unblemished skin and is uniform in color. The abdomen has a symmetric contour. There were symmetric movements caused associated with client’s respiration. The jugular veins are not visible. When nails pressed between the fingers (Blanch Test), the nails return to usual color in less than 4 seconds. Extremities The extremities are symmetrical in size and length. Muscles: The muscles are not palpable with the absence of tremors. They are normally firm and showed smooth, coordinated movements. Bones: There were no presence of bone deformities, tenderness and swelling. Joints: There were no swelling, tenderness and joints move smoothly. Performing a Comprehensive Health Assessment A comprehensive health assessment includes: • A complete medical history, A general survey, A complete physical assessment The medical history and the general survey were previously detailed. In this section, you will review the components of the complete physical assessment. Vital Signs The pulse, blood pressure, bodily temperature and respiratory rate are measured and documented. Assessment of the Thorax Inspection: The anterior and posterior thorax is inspected for size, symmetry, shape and for the presence of any skin lesions and/or misalignment of the spine; chest movements are observed for the normal movement of the diaphragm during respirations. Palpation: The posterior thorax is assessed for respiratory excursion and fremitus. Now is the best time to be the person you dream of becoming P a g e | 41 Percussion: For normal and abnormal sounds over the thorax Assessment of the Lungs Auscultation: The assessment of normal and adventitious breath sounds. Percussion: For normal and abnormal sounds. Normal breath sounds like vesicular breath sounds, bronchial breath sounds, bronchovesicular breath sounds are auscultated and assessed in the same manner that adventitious breath sounds like rales, wheezes, friction rubs, rhonchi, and abnormal bronchophony, egophony, and whispered pectoriloquy are auscultated, assessed and documented. Assessment of the Cardiovascular System Inspection: Pulsations indicating the possibility of an aortic aneurysm Auscultation: Listening to systolic heart sounds like the normal S1 heart sound and abnormal clicks, the diastolic heart sounds of S2, S3, S4, diastolic knocks and mitral valve sounds, all of which are abnormal with the exception of S2 which can be normal among clients less than 40 years of age. Assessment of the Peripheral Vascular System Inspection: The extremities are inspected for any abnormal color and any signs of poor perfusion to the extremities, particularly the lower extremities. While the client is in a supine position, the nurse also assesses the jugular veins for any bulging pulsations or distention. Auscultation: The nurse assesses the carotids for the presence of any abnormal bruits. Palpation: The peripheral veins are gently touched to determine the temperature of the skin, the presence of any tenderness and swelling. The peripheral vein pulses are also palpated bilaterally to determine regularity, number of beats, volume and bilateral equality in terms of these characteristics. Assessment of the Musculoskeletal System Inspection: The major muscles of the body are inspected by the nurse to determine their size, and strength, and the presence of any tremors, contractures, muscular weakness and/or paralysis. All joints are assessed for their full range of motion. The areas around the bones and the major muscle groups are also inspected to determine any areas of deformity, swelling and/or tenderness. Palpation: The muscles are palpated to determine the presence of any spasticity, flaccidity, pain, tenderness, and tremors. Assessment of the Neurological System Of all of the bodily systems that are assessed by the registered nurse, the neurological system is perhaps the most extensive and complex. Some of the terms and terminology relating to the neurological system and neurological system disorders that you should be familiar with include those below. Acalculia: Acalculia is the client's loss of ability to perform relatively simple mathematical calculations like addition and subtraction. Agnosia: Agnosia is defined as the loss of a client's ability to recognize and identify familiar objects using the senses despite the fact that the senses are intact and normally functioning. The different types of agnosia, as based on each of the five senses, are auditory agnosia, visual agnosia, gustatory agnosia, olfactory agnosia, and tactile agnosia. Agraphia: Agraphia, simply defined, is the Inability of the client to write. Agraphia is one of the four hallmark symptoms of Gerstmann's syndrome. The other symptoms of Gerstmann's syndrome are acalculia, finger agnosia, and an inability to differentiate between right and left. Alexia: Alexia, which is a type of receptive aphasia, occurs when the client is unable to process, understand and read the written word. This neurological disorder is also referred to as word blindness and optical alexia. Anhedonia: Anhedonia is a loss of interest in life experiences and life itself as the result of the neurological deficit. Anomia: Anomia is a lack of ability of the client to name a familiar object or item. Anosagnosia: Anosagnosia is characterized with the client's inability to perceive and have an awareness of an affected body part such as a paralyzed or missing leg. Anosagnosia is closely similar to hemineglect and hemiattention Anosdiaphoria: Anosdiaphoria is an indifference to one's illness and disability Aphasia: Aphasia includes expressive aphasia and receptive aphasia. Expressive aphasia is characterized by the client's inability to express their feelings and wishes to others with the spoken word; and receptive aphasia is the client's inability to understand the spoken words of others. Asomatognosi: Asomatognosia is the inability of the client to recognize one or more of their own bodily parts. Astereognosia: Astereognosia is the client's inability to differentiate among different textures with their sense of touch and also the inability of the client to identify a familiar object, like a button, with their tactile sensation. Now is the best time to be the person you dream of becoming P a g e | 42 Asymbolia: Asymbolia is the loss of the client's inability to respond to pain even though they have the sensory function to feel and perceive the pain. Asymbolia is also referred to as pain dissociation and pain asymbolia. Autotopagnosia: Autotopagnosia is the inability of the client to locate their own body parts, the body parts of another person, or the body parts of a medical model. Balint's syndrome: Balint's syndrome includes ocular apraxia, optic ataxia and simultanagnosia, which consist of impaired visual scanning, visusopatial ability and attention. Boston Diagnostic Aphasia Examination: The Boston Diagnostic Aphasia Examination is a standardized comprehensive assessment tool that assess and measures the client's degree of aphasia in terms of the client's perceptions, processing of these perceptions and responses to these perceptions while using problem solving and comprehension skills. Broca's aphasia: Broca's aphasia entails the client's lack of ability to form and express words even though the client's level of comprehension is intact. Color agnosia: Color agnosia reflects the client's lack of ability to recognize and name different colors. Conduction aphasia: Conduction aphasia is the client's lack of ability to repeat phrases and/or write brief dictated passages despite the fact that the client has intact speech abilities, comprehension abilities, and the ability to name familiar objects. Constructional apraxia: Constructional apraxia is the inability of the client to draw and copy simple shapes on paper. Dressing apraxia: Dressing apraxia occurs when the person is not able to appropriately dress oneself because of some neurological dysfunction. Dysgraphaesthesia: Dysgraphaesthesia impairs the client's ability to sense and identify a letter or number that is tactily drawn on the client's palm. Dysgraphia: Dysgraphia is similar to agraphia; however, dysgraphia is difficulty in terms of writing and agraphia is the client's complete inability to write. Environmental agnosia: Environmental agnosia is the lack of ability of the client to recognize familiar places, like the US Supreme Court, by looking at a photograph of it. Finger agnosia: Finger agnosia occurs when the person is not able to identify what finger is being touched by the person performing the neurological assessment. Geographic agnosia: Geographic agnosia is the lack of ability of the client to recognize familiar counties, like Canada or Mexico, when viewing a world map. Gerstmann's Syndrome: Gerstmann's Syndrome consists of dyscalculia or acalculia, finger agnosia, one sided disorientation and dysgraphia or agraphia. Hemiasomatognosia: Hemiasomatognosia is the neurological disorder that occurs when the client does not perceive one half of their body and they act in a manner as if that half of the body does not even exist. Homonymous hemianopsia: Homonymous hemianopsia occurs when the person has neurological blindness in the same visual field of both eyes bilaterally. Ideomotor apraxia: Ideomotor apraxia is a neurological deficit that affects the client's ability to pretend doing simple tasks of everyday living like brushing one's teeth. Misoplegia: Misoplegia is a hatred and distaste for an adversely affected limb. Motor alexia: Motor alexia occurs when the client is not able to comprehend the written word despite the fact that the client can read it aloud. Musical alexia: Musical alexia is a client's inability to recognize a familiar tune like "The National Anthem" or "Silent Night". Movement agnosia: Movement agnosia is a neurological deficit that is characterized with a client's lack of ability to recognize an object's movement. Ocular apraxia: Ocular apraxia is the neurological deficit that occurs when the person is no longer able to rapidly move their eyes to observe a moving object. Optic ataxia: Optic ataxia is characterized with the client's inability to reach for and grab an object. Phonagnosia: Phonagnosia is the client's lack of ability to recognize familiar voices such as those of a child or spouse. Prosopagnosia: Prosopagnosia is a lack of ability to recognize familiar faces, like the face of a spouse or child. Simultanagnosia: Simultanagnosia is a neurological disorder that occurs when the client is not able to perceive and process the perception of more than object at a time that is in the client's visual field. Somatophrenia: Somatophrenia occurs when the client denies the fact that their body parts are not even theirs, but instead, these body parts belong to another. The Two-Point Discrimination Test: This test measures and assesses the client's ability to recognize more than one sensory perception, such as pain and touch, at one time. Visual agnosia: Visual agnosia is the client's lack of ability to recognize and attach meaning to familiar objects.
Docsity logo



Copyright © 2024 Ladybird Srl - Via Leonardo da Vinci 16, 10126, Torino, Italy - VAT 10816460017 - All rights reserved